Mini Exam USPTO Questions

अब Quizwiz के साथ अपने होमवर्क और परीक्षाओं को एस करें!

Which of the following practices or procedures may be properly employed to overcome a rejection properly based only on AIA 35 USC 102(a)(2)? (A) Persuasively arguing that the claims are patentably distinguishable from the prior art. (B) Filing an affidavit or declaration under 37 CFR 1.132 showing that the reference invention is not by "another." (C) Filing an affidavit or declaration under 37 CFR 1.131 showing prior invention, if the reference is not a U.S. patent that either claims the same invention or claims an obvious variation of the subject matter in the rejected claim(s). (D) (A) and (C). (E) (A), (B) and (C).

(E) (A), (B) and (C).

or purposes of determining whether a request for continued examination is in accordance with proper USPTO rules and procedure, in which of the following situations will prosecution be considered closed? A The last Office action is a final rejection. B The last Office action is an Office action under Ex Parte Quayle. C A notice of allowance has issued following a reply to a first Office action. D The application is under appeal. E All of the above.

E All of the above.

A utility application filed in May 2016 has been prosecuted through a second action final rejection. In the final rejection some claims were allowed and other claims were finally rejected. Which of the following accords with the patent laws, rules and the procedures as related in the MPEP for a proper reply to a second action final rejection in the utility application? A An amendment canceling all rejected claims and complying with 37 CFR 1.116. B Only a Notice of Appeal. C The appropriate fee for a request for continued examination (RCE). D A continued prosecution application (CPA) under 37 CFR 1.53(d). E All of the above.

A An amendment canceling all rejected claims and complying with 37 CFR 1.116. NOT B because you nee a fee with it too

Claims 1 and 2 in a patent application state the following: Claim 1. An apparatus for sitting comprising: (i) a square shaped base member; (ii) four elongated members mounted to the bottom of the base member; and (iii) a circular back member mounted to the base member. Claim 2. An apparatus as in claim 1, further comprising a spring connected to the back member and to the base member.Which, if any, of the following claims fully supported by the specification and presented in the application, is in accordance with USPTO rules and procedure? A 3. An apparatus as in claim 1, wherein the base member is rectangularly shaped. B 3. An apparatus as in claim 2, wherein the wheels connected to each of the elongated members are plastic. C 3. An apparatus as in the preceding claims, further comprising a pressure-sensing device connected to the base member. D 3. An apparatus as in any of the preceding claims, in which the circular back member is wooden. E None of the above.

3. An apparatus as in any of the preceding claims, in which the circular back member is wooden.

Your client John K. Flier filed a provisional application on October 13, 2007 on an invention relating to an improved cup holder. The application was properly replaced by a non-provisional on September 23, 2008. While you were on vacation in Idaho, John called your office and insisted that an application in Mexico be filed immediately and the Mexican application was filed on September 30, 2008. John now asks you whether a Foreign Filing license should have been obtained prior to the filing in Mexico, and, if so, what should be done now. Your advice should be: A A Foreign Filing License was granted by the passage of six months following the filing of the provisional application. B A Foreign Filing License is not required for cup holders. C While a Foreign Filing License is technically required, the law is not enforced and can be ignored. D A retroactive Foreign Filing License is required. E The application should be abandoned.

A Foreign Filing License was granted by the passage of six months following the filing of the provisional application.

Which of the following statements concerning reliance by an examiner on common knowledge in the art, in a rejection under 35 USC 103 is correct? I. An examiner's statement of common knowledge in the art is taken as admitted prior art, if applicant does not seasonably traverse the well known statement during examination. II. Applicant can traverse an examiner's statement of common knowledge in the art, at any time during the prosecution of an application to properly rebut the statement. III. If applicant rebuts an examiner's statement of common knowledge in the art in the next reply after the Office action in which the statement was made, the examiner can never provide a reference to support the statement of common knowledge in the next Office action and make the next Office action final. A I. B II. C III. D I and II. E None of the above.

A I. An examiner's statement of common knowledge in the art is taken as admitted prior art, if applicant does not seasonably traverse the well known statement during examination.

In an application filed with an ADS, an inventor's oath or declaration must: A Identify the inventor by his or her legal name. B Identify the PTO-provided serial number of the application to which the oath or declaration is directed. C Include the mailing address where the inventor customarily receives mail, and residential address if an inventor lives at a location different from where the inventor customarily receives mail. D Acknowledge the inventor's duty to disclose to the Patent Office information material to patentability. E All of the above.

A Identify the inventor by his or her legal name.

Which of the following is true? A A claim to a process omitting a step in a disclosed process, where the step is disclosed in the specification to be essential to the invention, may not be properly rejected under 35 USC 112(a) for lack of enablement where the specification provides an enabling disclosure only for the process which includes the essential step. B Failure to disclose the best mode must rise to the level of active concealment or grossly inequitable conduct in order to support a rejection under 35 USC 112(a). C A claim failing to interrelate essential elements of the invention, as defined by the applicant in the specification, where the interrelation is critical to the invention may be properly rejected under 35 USC 112(b) for failure to properly point out and distinctly claim the invention. D Where the best mode contemplated by the inventor at the time of filing the application is not disclosed, a proposed amendment adding a specific mode of practicing the invention would not be new matter. E The best mode requirement is the same as the enablement requirement of the first paragraph of 35 USC 112.

A claim failing to interrelate essential elements of the invention, as defined by the applicant in the specification, where the interrelation is critical to the invention may be properly rejected under 35 USC 112(b) for failure to properly point out and distinctly claim the invention.

Your new client Mary Williams has come to you because she is dissatisfied with her previous agent. She has provided you with a copy of her application as filed on June 4, 2011. She is uncertain what, if anything, might have been done or received by her previous attorney. She said that her previous attorney hasn't returned her phone calls for more than a year and his assistant told her that he was on an extended trip to Tibet for religious reasons.It turns out that Mary's application filed in June of 2011 was abandoned early in 2013, and a proper continuation filed under Rule 1.53(b). Before the parent was abandoned a proper claim for foreign priority was made based upon an application filed in Canada on June 11, 2010. A certified copy of the Canadian application was filed on December 14, 2011. What is the minimum action you must take to preserve the foreign priority in the continuation application? A No action needs to be taken because a proper claim was made in the parent application. B A proper claim must be made in the continuation for the benefit of the Canadian priority and a certified copy of the Canadian application must be filed in the continuation. C A proper claim must be made in the continuation for the benefit of the Canadian priority, but a non-certified copy of the Canadian application can be filed in the continuation, because a certified copy is already in the parent file. D A claim must be made in the continuation for the benefit of the Canadian priority and the application containing the certified copy of the Canadian application must be identified. E A proper claim must be made in the continuation for the benefit of the Canadian priority and the Canadian Patent Office must be requested to send a certified copy of the Canadian application to the USPTO.

A claim must be made in the continuation for the benefit of the Canadian priority and the application containing the certified copy of the Canadian application must be identified.

Which of the following is not in accordance with the provisions of 35 USC 115 (Oath of applicant)? A The applicant shall make oath (or declaration) that he believes himself to be an original inventor of the process, machine, manufacture, or composition of matter, or improvement thereof, for which he solicits a patent. B In the oath or declaration, the applicant must state that the application is filed with their consent. C An oath may be made before any person within the United States authorized by law to administer oaths. D An oath executed in a foreign country must be properly authenticated. E A declaration which accompanies a patent application must state on the document a warning that willful false statements and the like are punishable by fine or imprisonment or both under 18 USC 1001, and the declaration must be notarized.

A declaration which accompanies a patent application must state on the document a warning that willful false statements and the like are punishable by fine or imprisonment or both under 18 USC 1001, and the declaration must be notarized. DECLARATIONS DO NOT NEED TO BE NOTARIZED!

With the exception that under 37 CFR 1.53 an application for patent may be assigned a filing date without payment of the basic filing fee, USPTO fees and charges payable to the USPTO requesting any action by the Office for which a fee or charge is payable, are required to be paid, in accordance with the MPEP and USPTO rules and procedure: A in advance, that is, at the time of requesting any action. B upon written notice from the USPTO. C within 20 days of requesting any action. D by the end of the fiscal year. E there are no fees.

A in advance, that is, at the time of requesting any action.

Paprika is a known product. A patent application discloses a composition which is made by subjecting paprika to processing steps X, Y and Z. The composition is disclosed to be useful in treating cancer. The application was filed June 1, 2016. A reference published May 1, 2015 discloses a food product made by subjecting paprika to processing steps X, Y and Z. The reference does not disclose that the resulting composition has any properties that would make it useful for treating cancer. In accordance with USPTO rules and procedures set forth in the MPEP, which of the following claims is not anticipated by the reference? A A composition made by the process of subjecting paprika to processing steps X, Y and Z, wherein the composition is effective for treating cancer. B A composition for treating cancer, made by the process of subjecting paprika to processing steps X, Y and Z. C A method of making a cancer-treating composition, comprising subjecting paprika to processing steps X, Y and Z. D A method of treating cancer, comprising administering an effective amount of a composition made by subjecting paprika to processing steps X, Y and Z. E All of the above.

A method of treating cancer, comprising administering an effective amount of a composition made by subjecting paprika to processing steps X, Y and Z. to avoid anticipation we can't claim the substance (prior art), must claim the method

Recommend which of the following rejections under 35 USC 102 in a Exparte reexamination proceeding is in accordance with the patent laws, rules and procedures as related in the MPEP for pre-03/16/13 filed patents. A A rejection under 35 USC 102(a) based on an affidavit that the invention was known or used by others before the invention thereof by the applicant for patent. B A rejection under 35 USC 102(b) based on an affidavit that the invention was in the public use in this country more than one year prior to the date of the application for a patent in the United States. C A rejection under 35 USC 102(e) that the invention was described in a patent by another filed in the United States before the invention thereof by the patent applicant. D A rejection under 35 USC 102(f) based on an affidavit that the applicant did not himself invent the subject matter sought to be patented. E A rejection under 35 USC 102(b) that the invention was on sale in this country, more than one year prior to the date of the application for patent in the United States.

A rejection under 35 USC 102(e) that the invention was described in a patent by another filed in the United States before the invention thereof by the patent applicant. [r]ejections on prior art in reexamination proceedings may only be made on the basis of prior patents or printed publications

Which of the following is not in accordance with the provisions of the MPEP Chapter 600? A A request for a refund must be filed within two years from the date the fee was paid or, in the case of a fee paid by mistake, within one year from the time the error was discovered. B A change of purpose after the payment of a fee, such as when a party desires to withdraw a patent filing for which the fee was paid, including an application, an appeal or a request for an oral hearing, will not entitle a party to a refund of such fee. C The Office will not refund amounts of twenty-five dollars or less, unless a refund is specifically requested. D Any refund of a fee paid by credit card will be by a credit to the credit card account to which the fee was charged. E When a fee is paid where no fee is required, this is considered to be a fee paid by mistake.

A request for a refund must be filed within two years from the date the fee was paid or, in the case of a fee paid by mistake, within one year from the time the error was discovered.

Which of the following statements about foreign priority is false? A An inadvertent failure without deceptive intent to claim or perfect a claim for foreign priority can be cured by seeking a reissue. B The priority period for design applications is six months. C A translation submitted to establish foreign priority rights of an application not in English need not be sworn or in the form of a declaration. D A translation must be submitted to perfect priority unless the certified copy is in English. E Rather than submitting another certified copy of a priority document reference can be made to a document previously filed in another U.S. application.

A translation must be submitted to perfect priority unless the certified copy is in English.

A registered practitioner filed a patent application naming Sam as the sole inventor without an executed declaration under 37 CFR 1.63. The USPTO mailed a Notice to File Missing Parts dated January 3, 2016. The Notice to File Missing Parts set a two-month period for reply. Which of the following statements is in accordance with proper USPTO rules and the procedure set forth in the MPEP?I I. Submit an appropriate reply to the Notice to File Missing Parts by filing, on August 3, 2016, a declaration under 37 CFR 1.63 executed by Sam, accompanied by a petition under 37 CFR 1.136(a) for an extension of five months, and the fee set forth in 37 CFR 1.17(a). II. In no situation can any extension requested by the practitioner carry the date on which a reply is due to the Notice to File Missing Parts beyond Monday, July 3, 2016 .III. An appropriate reply by the practitioner to the Notice to File Missing Parts is to file, on August 3, 2016 a declaration under 37 CFR 1.63 executed by Sam, accompanied by a petition under 37 CFR 1.136(b). AI BII CIII DI and III ENone of the above.

AI (B) is incorrect because a Notice to File Missing Parts of an Application is not identified on the Notice as a statutory period subject to 35 U.S.C. § 133. The 2-month time period for reply to A Notice to File Missing Parts of an Application is not identified on the Notice as a statutory period subject to 35 U.S.C. 133. Thus, extensions of time of up to 5 months under 37 CFR 1.136(a), followed by additional time under 37 CFR 1.136(b), when appropriate, are permitted. SO YOU CAN GET 7 months

Applicant properly appealed the primary examiner's final rejection of the claims to the PTAB (Board). Claims 1 to 10 were pending in the application. The examiner did not reject the subject matter of claims 7 to 10, but objected to these claims as being dependent on a rejected base claim. Claim 1 was the sole independent claim and the remaining claims, 2 through 10, were either directly or indirectly dependent thereon. After a thorough review of Appellant's brief and the examiner's answer, the Board affirmed the rejection of claims 1 to 6. In accordance with the USPTO rules and the procedures set forth in the MPEP, which of the following is the appropriate action for the examiner to take upon return of the application to his jurisdiction when the time for appellant to take further action has expired? A Abandon the application since the Board affirmed the rejection of independent claim 1. B Convert the dependent claims 7 to 10 into independent form by examiner's amendment, cancel claims 1 to 6, and allow the application. C Mail an Office action to applicant setting a 1-month time limit in which the applicant may rewrite dependent claims 7 to 10 in independent form. If no timely reply is received, the examiner should amend the objected to claims, 7 to 10, and allow the allow the application. D Mail an Office action to applicant with a new rejection of claims 7 to 10 based on the Board's decision. E No action should be taken by the examiner since the Board affirmed the rejection of independent claim 1, the application was abandoned on the date the Board decision was mailed.

Abandon the application since the Board affirmed the rejection of independent claim 1. Why is it not E? E) is not correct because the mailing ofa Board decision does not abandoned an application

Applicant filed an application containing a claim directed to a polishing wheel coated with diamond grit particles. The application discloses, but does not claim, a diamond grit particle size of 5-7 microns. The examiner rejected the claim under 35 USC 102 as being anticipated by a U.S. patent which disclosed as its invention a polishing wheel in accordance with the claim of the application but coated with glass grit particles instead of diamond grit particles. The applied patent, which issued more than 1 year prior to the effective filing date of the application, also disclosed that diamond grit particles were known for coating on polishing wheels but were inferior to glass grit particles because they were more expensive and did not adhere as well to the polishing wheel. The applied patent disclosed a grit particle size of 50-100 microns. Which of the following timely taken courses of action would comply with the patent laws, rules and procedures as related in the MPEP for overcoming the rejection? A Argue that the patent teaches away from the use of a diamond grit particle coating on a polishing wheel and thus does not teach the claimed invention. B File a declaration under 37 CFR 1.132 showing unexpected results using diamond grit rather than glass grit. C Antedate the applied patent by filing a declaration under 37 CFR 1.131 showing that applicant invented the claimed subject matter prior to the effective date of the applied patent. D Argue the applied patent is nonanalogous art. E Amend the claim by adding a limitation that the diamond grit particle size is 5-7 microns, and arguing that the claimed invention differs from applied patent by limited the diamond grit particle size to 5-7 microns.

Amend the claim by adding a limitation that the diamond grit particle size is 5-7 microns, and arguing that the claimed invention differs from applied patent by limited the diamond grit particle size to 5-7 microns. Why not C? YOU CANNOT FILE AN AFFADAVIT/DECLARATION to swear behind something that has occurred more than one year before your filing date! it is E because "A claim is anticipated only if each and every element as set forth in the claim is found, either expressly or inherently described, in a single prior art reference."

With respect to the examiner's burden in making an enablement rejection under 35 USC 112(a), which of the following statements is or are in accordance with the patent laws, rules and procedures as related in the MPEP? (1) The examiner may properly make an enablement rejection before construing the claims. (2) The examiner has the initial burden to establish a reasonable basis to question the enablement provided for the claimed invention. (3) The examiner need not give reasons for the uncertainty of the enablement when there is no evidence of operability beyond the disclosed embodiments. A Statement (1) only B Statement (2) only C Statement (3) only D Statements (1) and (2) E Statements (1) and (3)

B Statement (2) only

In accordance with the patent laws, rules and procedures as related in the MPEP , which of the following paper is precluded from receiving the benefit of a certificate of mailing or transmission under 37 CFR 1.8? A An amendment, replying to an Office action setting a period for reply, transmitted by mail with a certificate of mailing to the USPTO from a foreign country. B An amendment, replying to an Office action setting a period for reply, transmitted by facsimile with a certificate of transmission to the USPTO from a foreign country. C An information disclosure statement (IDS) under 37 CFR 1.97 and 1.98 transmitted after the first Office action. D A request for continued examination (RCE) under 37 CFR 1.114. E An appeal brief.

An amendment, replying to an Office action setting a period for reply, transmitted by mail with a certificate of mailing to the USPTO from a foreign country. "The Certificate of Mailing procedure does not apply to papers mailed in a foreign country."

Applicant files a patent application in Japan on January 5, 2000. Applicant files a PCT international application designating the United States on January 5, 2001, based on the Japanese application. The international application is published in English on July 5, 2001. The international application enters the national stage in the United States on September 5, 2001. The USPTO publishes the application on June 6, 2002. The application issues as a United States patent on December 3, 2002. What is its earliest possible 35 USC 102(e) prior art date for the application published by the United States, in view of the amendment to Title 35 by the American Inventors Protection Act of 1999 and the Intellectual Property and High Technology Technical Amendments Act of 2002? A January 5, 2000. B January 5, 2001. C July 5, 2001. D June 6, 2002. E December 3, 2002.

B January 5, 2001. a US published application of a national stage of an international application filed on or after November 29, 2000 has a prior art effect as of its international filing date, if the internationalapplication designated the United States, and wa s published in English FOT/DUSE gives you a prior art date of the international filing date!

A patent application includes the following Claim 1:Claim 1. A method of making an electrical device comprising the steps of: (i) heating a base made of carbon to a first temperature in the range of 1875 degrees C to 1925 degrees C; (ii) passing a first gas over said heated base, said first gas comprising a mixture of hydrogen, SiCl4, phosphorus, and methane, whereby said first gas decomposes over said heated base and thereby forms a first deposited layer of silicon, phosphorus and carbon on said heated base; (iii) heating said base having said deposited layer to a second temperature of approximately 1620 degrees C; and (iv) passing a second gas over said base heated to said second temperature, said second gas consisting of a mixture of hydrogen, SiCl4, AlCl3, and methane, whereby said second gas decomposes over said heated base to form a second deposit layer adjacent said first layer, said second layer comprising silicon, aluminum and carbon. Assuming proper support in the specification, which of the following claims, if presented in the same application, is a proper claim in accordance with the USPTO rules and the procedures set forth in the MPEP? A Claim 2. The method of claim 1, wherein said first temperature is in the range of 1800 degrees C to 2000 degrees C. B Claim 3. The method of claim 1, wherein said first gas further comprises an inert gas. C Claim 4. The method of claim 1, wherein said second gas further comprises Argon. D Claim 5. The method of claim 1, wherein said first gas is an inert gas such as Argon. E Claim 6. The method of claim 1, wherein said second gas consists of a mixture of hydrogen, SiCl4 and AlCl3 only.

B Claim 3. The method of claim 1, wherein said first gas further comprises an inert gas. Answers (A) and (E) are incorrect because they improperly seek to broaden the parent claim. 37 CFR § 1.75(c). Answer(A) broadens the range by going below the stated limit. Answer (E) broadens by trying to remove a recited component of the second gas, and covering subject matter that is not covered by the parent claim. "comprises" language means you can only take the parent "comprise" language and add on to that with further comprising... whereas "consisting of" language means you can only add on to the parent claim "consisting of" and add on more "consisting of.." language it's also not C because (C) is incorrect because claim 1 uses the close ended claim term "consists" in connection with the second gas, which precludes the addition of further components to the second gas in claim 4.

appellant believes that an examiner's answer, mailed on November 1, 2016, contains a new ground of rejection, but it is not labeled as such. If an amendment or new evidence is needed to overcome the new ground of rejection, what is the best course of action the appellant should take in accordance with the USPTO rules and the procedures set forth in the MPEP? A File a reply brief bringing the new ground of rejection to the attention of the Board of Patent Appeals and Interferences and pointing out that 37 CFR 41.39(a)(2) prohibits entry of the new ground of rejection. B Ignore the new ground of rejection. C File a reply brief arguing the merits of the new ground of rejection. D File an amendment or new evidence to overcome the new ground of rejection. E File a timely petition pursuant to 37 CFR 1.181 seeking review such that the new ground of rejection in the answer is so designated, after efforts to persuade the examiner to reopen prosecution or remove the new ground of rejection are unsuccessful.

B Ignore the new ground of rejection. not sure if this is actually correct - the answer key says that you need to petition this...double check!

A patent application has claims 1-10 pending. Claims 1 and 7 are independent claims. Claims 2-6 depend directly from claim 1 while claims 8-10 depend directly from claim 7. Claims 1-10 have been twice rejected by the primary examiner under 35 USC 103 as being unpatentable over Smith patent in view of Jones patent. The applicant has appealed the rejection to the PTAB. In the brief under the "Grounds of Rejection to be Reviewed" section, appellant states that each of the claims is separately patentable. In the arguments section of the brief, appellant separately argues only claims 1, 4 and 6. In the examiner's answer, the examiner disagrees with appellant's claim grouping because all the claims present a similar issue of patentability. The examiner states that the claims all stand or fall together as a single group. In accordance with the patent laws, rules and procedures as related in the MPEP, which claim(s) must the Board consider separately on the merits? A The Board must consider each of claims 1-10 separately on the merits. B The Board must only consider claims 1, 4 and 6 separately on the merits. C The Board must only consider claim 1 separately on the merits. D The Board must consider claim 1 and claim 7 separately on the merits as representative of all the claims on appeal. E The Board must determine which claim is representative of all the claims on appeal and consider only that claim separately on the merits.

B The Board must only consider claims 1, 4 and 6 separately on the merits. The appellant must (1) state that the claims do not stand or fall together and (2) present arguments why the claims subject to the same rejection are separately patentable. Since the appellant here has only performed the two affirmative acts with respect to claims 1, 4 and 6,these are the claims that the Board must consider separately for patentability. The examiner has no input on the grouping of claims.

In accordance with the patent laws, rules and procedures as related in the MPEP, which of the following statements regarding claim interpretation during patent prosecution is incorrect? AA claim is to be given its broadest reasonable interpretation in light of the supporting disclosure in the specification. BBecause a claim is read in light of the specification, the claim may properly be narrowed by interpreting it as including elements or steps disclosed in the specification but not recited in the claim. CIf an applicant does not define a claim term in the specification, that term is given its ordinary meaning in the art. DWhen an explicit definition of a claim term is provided in an applicant's specification, that definition controls the interpretation of the term as it is used in the claims. EMeans plus function language in claims which defines the characteristics of a machine or manufacture includes only the corresponding structures or materials disclosed in the specification and equivalents thereof.

BBecause a claim is read in light of the specification, the claim may properly be narrowed by interpreting it as including elements or steps disclosed in the specification but not recited in the claim. **LOOKING FOR WHAT IS INCORRECT

A registered practitioner filed a utility application on February 11, 2016. On April 4, 2016, the practitioner filed an information disclosure statement (IDS) in the application. The practitioner received a notice of allowance dated January 3, 2017 soon after it was mailed. When discussing the application with the practitioner on January 21, 2017, and before paying the issue fee, the client notices for the first time that a reference, which is one of many patents obtained by the client's competitor, was inadvertently omitted from the IDS. The client has been aware of this reference since before the application was filed. The client is anxious to have this reference appear on the face of the patent as having been considered by the USPTO. Which of the following actions, if taken by the practitioner, would not be in accord with the patent law, rules and procedures as related by the MPEP? A Before paying the issue fee, timely file an IDS citing the reference, along with the certification specified in 37 CFR 1.97(e), and any necessary fees. B Within three months of the mail date of the notice of allowance, without paying the issue fee, timely file a Request for Continued Examination (RCE) under 37 CFR 1.114, accompanied by the fee for filing an RCE, and an IDS citing the reference. C Within three months of the mail date of the notice of allowance, without paying the issue fee, timely file a continuing application under 37 CFR 1.53(b), an IDS citing the reference, and any necessary fees. D After paying the issue fee, timely file a petition to withdraw the application from issue to permit the express abandonment of the application in favor of a continuing application, a continuation application under 37 CFR 1.53(b), an IDS citing the reference, and any necessary fees. E After paying the issue fee, timely file a petition to withdraw the application from issue to permit consideration of a Request for Continued Examination (RCE) under 37 CFR 1.114, the fee for filing an RCE, and an IDS citing the reference.

Before paying the issue fee, timely file an IDS citing the reference, along with the certification specified in 37 CFR 1.97(e), and any necessary fees. The statement specified in 37 CFR § 1.97(e) requires that the practitioner certify, after reasonable inquiry, that no item of information contained in the IDS was known to any individual designated in 37 CFR § 1.56(c) more than three months prior to the filing of the information disclosure statement. Why not E? a petition to withdraw the application from issue can be filed after payment of the issue fee to permit consideration of a Request for Continued Examination (RCE)

Application A was filed after November 29, 2016. Reference X and application A were commonly owned at the time the invention of application A was made, and later filed. In accordance with the patent laws, rules and procedures as related in the MPEP the prior art exclusion of 35 USC 102(c) can be properly invoked to obviate which of the following rejections? A A rejection under 35 USC 102 based on reference X, if reference X is prior art under 35 USC 102(a)(1). B A double patenting rejection based on reference X, if reference X is available as prior art under 35 USC 102. C A rejection under 35 USC 103 based on reference X, if reference X is available as prior art only under 35 USC 102(a)(2). D (B) and (C). E All of the above.

C A rejection under 35 USC 103 based on reference X, if reference X is available as prior art only under 35 USC 102(a)(2).

John filed a nonprovisional patent application in the USPTO claiming two distinct inventions, a combination and a subcombination. At the time of filing the nonprovisional application, he recorded an assignment of all right, title, and interest in the inventions claimed in the application to ABC Corporation. In the first Office action, the examiner required restriction, and John elected the combination. A year later, during the pendency of the nonprovisional application, John filed a divisional patent application claiming the subcombination. At the time of filing the divisional application, John assigned all right, title, and interest in the inventions claimed in the divisional application to XYZ Corporation, and the latter party recorded the assignment within three months of the assignment. Following recordation of the assignment to XYZ Corporation, which of the following statements is false? A The Office should treat John as having no ownership rights in the combination. B The Office should treat John as having no ownership rights in the subcombination. C ABC Corporation has no ownership rights in the subcombination. D XYZ Corporation has no ownership rights in the combination .E XYZ Corporation has no ownership rights in the subcombination.

C ABC Corporation has no ownership rights in the subcombination. ABC rights from parent application override XYZ

In accordance with the USPTO rules and the procedures of the MPEP, which of the following is true? A If after the filing of a reissue application no errors in the original patent are found, a reissue patent will be granted on the reissue application noting no change, and the original patent will be returned to the applicant. B In order to add matter not previously found in the patent, a continuation-in-part reissue application must be filed. C In a reissue application, additions and deletions to the original patent should be made by underlining and bracketing, respectively, except for changes made in prior Certificates of Correction and disclaimer(s) of claims under 37 CFR 1.321(a). D A dependent claim may be broadened in a reissue application only in the first two years of the enforceable life of the patent. E (A), (B), and (C).

C In a reissue application, additions and deletions to the original patent should be made by underlining and bracketing, respectively, except for changes made in prior Certificates of Correction and disclaimer(s) of claims under 37 CFR 1.321(a). NOTE: not B because you can't add new matter in a reissue!!

The primary examiner has rejected claims 1-10 under 35 USC 103 as being unpatentable over the Smith patent in view of the Jones reference. Appellant properly argues that there is no motivation or KSR rationale to combine the teachings of Smith and Jones. The examiner repeats the rejection of claims 1-10 as being "unpatentable over Smith in view of Jones." The examiner additionally cites a patent to Brown that was necessary to provide motivation for combining the teachings of Smith and Jones. The examiner does not list Brown in the statement of the rejection. Appellant timely appeals to the PTAB, and files a proper appeal brief. The examiner files an examiner's answer addressing the rejection of claims 1-10 under 35 USC 103 as being unpatentable over Smith in view of Jones, and cites Brown in the argument as providing motivation to combine Smith and Jones. In accordance with the patent laws, rules and procedures as related in the MPEP, what will be the most proper decision of the Board? A The Board will affirm the rejection based on Smith and Jones only. B The Board will affirm the rejection based on Smith, Jones and Brown. C The Board will reverse the rejection based on Smith and Jones only. D The Board will reverse the rejection based on Smith, Jones and Brown. E None of the above.

C The Board will reverse the rejection based on Smith and Jones only. Ift he claimed invention is rendered obvious by Smith in view of Jones and Brown, the statement of rejection must include all three references. Reliance on Brown to support the rejection is a different rejection from a rejection relying only on Smith in view of Jones

A patent application filed in the USPTO contains the following three original claims, including product by process Claim 3: Claim 1. A method for making an Ethernet cable comprising the steps of A, B and C. Claim 2. The method of claim 1, further characterized by the step of D. Claim 3. The Ethernet cable as in any one of the preceding claims. In the first Office action, the examiner objects to Claim 3 as being an improper dependent claim and requires cancellation of the claim. Following proper USPTO practices and procedures, which of the following replies best overcomes the examiner's objection and provides the client with the broadest patent protection? A Amend Claim 3 to read: "The Ethernet cable as made by the process set forth in claims 1-2." B Cancel Claim 3. Add Claim 4, which reads: "An Ethernet cable made by a process comprising the steps of A, B and C." Add Claim 5, which reads: "An Ethernet cable made by a process comprising the steps of A, B, C and D." C Cancel Claim 3 and add Claim 4, which reads: "An Ethernet cable made by a process comprising the steps of A, B and C." D Cancel Claim 3 and add Claim 4, which reads: "An Ethernet cable made by a process comprising the steps of A, B, C and D." E Cancel Claim 3.

Cancel Claim 3. Add Claim 4, which reads: "An Ethernet cable made by a process comprising the steps of A, B and C." Add Claim 5, which reads: "An Ethernet cable made by a process comprising the steps of A, B, C and D."

A patent application filed in the USPTO contains the following dependent claim: Claim 2. The method of Claim 1, further consisting of the step of cooling the mixture to a temperature of 32 degrees F.Following proper USPTO practices and procedures, from which of the following claims does Claim 2 not properly depend? A Claim 1. A method of making liquid compound A consisting of the steps of mixing equal quantities of material C and material D in a beaker and heating the mixture to a temperature of 212Ã F. B Claim 1. A method of making liquid compound A comprising the steps of mixing equal quantities of material C and material D in a beaker and heating the mixture to a temperature of 212Ã F. C Claim 1. A method of making liquid compound A including the steps of mixing equal quantities of material C and material D in a beaker and heating the mixture to a temperature of 212Ã F. D Claim 1. A method of making liquid compound A characterized by the steps of mixing equal quantities of material C and material D in a beaker and heating the mixture to a temperature of 212Ã F. E (C) and (D).

Claim 1. A method of making liquid compound A consisting of the steps of mixing equal quantities of material C and material D in a beaker and heating the mixture to a temperature of 212Ã F. The phrase "consisting of" excludes any step not specified in the claim. MPEP § 2111.03. Thus, a claim that depends from a claim which "consists of" the recited steps cannot add a step. Id. Here, the dependent claim adds the step of cooling. if your claim 1 says "consisting of.." you CANNOT add any more steps (i.e.cannot add the step of cooling!) But if claim one said "comprising" then you could indeed add on a step, such as cooling

A patent specification discloses a personal computer comprising a microprocessor and a random access memory. There is no disclosure in the specification of the minimum amount of storage for the random access memory. In the disclosed preferred embodiment, the microprocessor has a clock speed of 100-200 megahertz. Claims 9 and 10, presented below, are original claims in the application. Claim 11, presented below, was added by amendment after an Office action. 9. A personal computer comprising a microprocessor and a random access memory that includes at least 1 gigabyte of storage. 10. The personal computer of Claim 9, wherein the microprocessor has a clock speed of 100-200 megahertz. 11. The personal computer of Claim 10, wherein the random access memory is greater than 1/2 gigabyte of storage. Which of the following statements is or are in accord with the patent laws, rules and procedures as related in the MPEP regarding the respective claims under the fourth paragraph of 35 USC 112? A Claim 9 is a proper independent claim, and Claims 10 and 11 are proper dependent claims. B Claim 9 is a proper independent claim, and Claims 10 and 11 are improper dependent claims. C Claim 9 is an improper independent claim, and Claims 10 and 11 are improper dependent claims. D Claim 9 is an improper independent claim, and Claims 10 and 11 are proper dependent claims. E Claim 9 is a proper independent claim, Claim 10 is a proper dependent claim, and Claim 11 is an improper dependent claim.

Claim 9 is a proper independent claim, Claim 10 is a proper dependent claim, and Claim 11 is an improper dependent claim. Claim 9, though broad, is supported by the specification. laim 10 is aproper dependent claim because it depends from and further restricts the scope of a precedingclaim. 37 CFR § 1.75(c). Claim 11 is an improper dependent claim because it is inconsistentwith and does not further limit the scope of claim 10. Claim 11 is not further limiting! Claim 10, depending on Claim 9, has a 1gigabyte memory minimum, whereas Claim 11 redefines the minimum memory by setting alower minimum of ½ gigabyte.

In accordance with USPTO rules and the procedure set forth in the MPEP, which one of the following is not required for a complete provisional application filed in the USPTO? A A specification. B A drawing, if necessary, as prescribed by 35 USC 113. C An application fee. D A claim. E A cover sheet complying with the rule.

D A claim. NOTE: it is a PROVISIONAL - provisionals do not have claims!!

Your new client Mary Williams has come to you because she is dissatisfied with her previous agent. She has provided you with a copy of her application as filed on June 4, 2011. She is uncertain what, if anything, might have been done or received by her previous attorney. She said that her previous attorney hasn't returned her phone calls for more than a year and his assistant told her that he was on an extended trip to Tibet for religious reasons.Mary was, in fact, not the only inventor listed on the parent application. Her ex-husband George was listed as a co-inventor. He made a small contribution, which was set forth in one claim - claim 29. George is now demanding that he receive half of all monies received. Mary wants George out of her life and out of her patent application. Which of the following is true? A George must remain as a co-inventor because inventorship can be changed only if a mistake has been made. B George may be deleted as a co-inventor under 37 CFR 1.48(a) if his contribution is no longer being claimed. C George cannot properly be a co-inventor because George made a contribution to only one claim. D A second continuation must be filed to delete George as an inventor. E An amendment deleting an inventor under 37 CFR 1.48(b) must be signed by the inventor deleted.

D A second continuation must be filed to delete George as an inventor. Remember - can't just delete inventors. a New ADS and fee must be provided

Mary wants to file a new application directed to an improvement and she wants this new application to expire as far in the future as possible, while costing the least amount of money to file. This improvement was fully described in a magazine, which was published almost a year ago. The most appropriate action would be: A Prepare a continuation-in-part application to replace the continuation now on file. B Prepare and file a new non-provisional application. C Advise Mary to delay filing the application on the improvement until there is infringement. D File a provisional application before the one year anniversary of the publication. E Amend the continuation to include the improvement.

D File a provisional application before the one year anniversary of the publication.

Xavier files a complete first reply exactly 10 weeks after the mailing date of a final Office action that sets a 3 month shortened statutory period for reply. An Examiner's Amendment is necessary for the purpose of placing the application in condition for allowance. Which of the following statements is true? A If Xavier gives authorization for the Examiner's Amendment exactly 2 months after his reply, the application will be allowed. B Authorization for the Examiner's Amendment may be made at any time within 6 months of Xavier's reply to avoid abandonment of the application. C Unless Xavier gives authorization for the Examiner's Amendment within the 3 months shortened statutory period for reply, the application will be abandoned. D If Xavier gives authorization for the Examiner's Amendment exactly 2 months after his reply, the application will be abandoned unless accompanied by a proper petition and fee for an extension of time. E Abandonment of the application will be avoided if Xavier gives authorization for the Examiner's Amendment any time within 6 months of the mail date of a final Office action. No extension of time need be filed if Xavier gives the authorization between 3 months and 6 months after the Office action.

D If Xavier gives authorization for the Examiner's Amendment exactly 2 months after his reply, the application will be abandoned unless accompanied by a proper petition and fee for an extension of time. you must give authorization in less than two months, otheriwse you need to petition for more time within the staturory period (3 months) otherwise it will be abandoned

Prosecution before the primary examiner results in the rejection of claim 1. Claim 2 was objected to as being allowable except for its dependency from claim 1. Independent claim 3 has been allowed. The rejection of claim 1 is properly appealed to the PTAB. The Board properly affirms the rejection of claim 1. Appellant has filed no response to the decision of the Board, the appellant has taken no action, and the time for filing an appeal to the court or a civil action has expired. In accordance with the patent laws, rules and procedures as related in the MPEP, which of the following actions is the most appropriate response by the examiner? A The examiner should hold the application abandoned. B The examiner should cancel claim 1, convert dependent claim 2 into independent form by examiner's amendment, and allow the application. C The examiner should set a 1-month time limit in which appellant may rewrite the dependent claim in independent form. D The examiner should cancel claims 1 and 2 and allow the application with claim 3 only. ENone of the above.

D The examiner should cancel claims 1 and 2 and allow the application with claim 3 only. answers A, B, and C only occur if no claims stand allowed!

Which of the following is in accordance with the provisions in the MPEP? A In order to correct inventorship in a nonprovisional application where the statement of the lack of deceptive intent is not available from an inventor to be added, a petition under 37 CFR 1.181 may be properly filed. B If a person A learns that a patent application has been filed by person B without naming A as coinventor, A may file in the USPTO a petition that protests inventorship and directs B to add A's name as a coinventor to the patent application. C If the application is involved in an interference, and a petition under 37 CFR 1.48 is filed to correct inventorship, the PTAB will remand the case to the primary examiner for consideration of the petition to ensure that a search of the relevant prior art is performed. D When a submission under 37 CFR 1.130 is attempted by the practitioner, the acceptance decision will be made by the Technology Center Director. E None of the above.

D When a submission under 37 CFR 1.130 is attempted by the practitioner, the acceptance decision will be made by the Technology Center Director. E None of the above. **KEY says D or E are both accepted

A patent application filed in the USPTO claims a nylon rope coated with element E for the purpose of preventing breakage of the rope. In the first Office action, the examiner rejects the claim as obvious over P in view of a trade journal publication, T. P teaches a nylon rope coated with resin for the purpose of making the rope waterproof. T teaches a nylon tent fabric coated with element E for the purpose of making the tent waterproof, and suggests the use of element E for making other nylon products waterproof. Following proper USPTO practices and procedures, the combination of P and T: A cannot support a prima facie case of obviousness because T lacks a suggestion to combine with P for the purpose of preventing breakage in nylon rope. B cannot support a prima facie case of obviousness because P lacks a suggestion to combine with T for the purpose of preventing breakage in nylon rope. C cannot support a prima facie case of obviousness because T only contains a suggestion to combine with P for the purpose of waterproofing nylon rope. D can support a prima facie case of obviousness, even though T only contains a suggestion to combine with P for the purpose of waterproofing nylon rope. E can support a prima facie case of obviousness because the applicant is always under an obligation to submit evidence of non-obviousness regardless of whether the examiner fully establishes a prima facie case of obviousness.

D. can support a prima facie case of obviousness, even though T only contains a suggestion to combine with P for the purpose of waterproofing nylon rope. It is not necessary in order to establish a prima facie case of obviousness...that there be a suggestion or expectation from the prior art that the claimed [invention] will have the same or a similar utility as one newly discovered by the applicant. Therefore, the utility does not have to be the same, but the actions must be the same. Here, we already are using both a nylon rope and element E, the two things the inventor is trying to claim. it is not necessary that the prior art suggest the combination to achieve the same advantage or result discovered by applicant

Herman and June Dulets are siblings and are listed as the co-inventors of a U.S. patent application now on file in 2013. They have just told you that Herman solely conceived of what was claimed and June reduced the invention to practice. You should advise them: A The inventorship is correct. B If anyone ever asks, tell them that both siblings are in fact co-inventors. C You must file a petition under 37 CFR 1.48(a) to correct inventorship by deleting June and satisfy the other requirements of the section. D You must file a petition under 37 CFR 1.48(b) to correct inventorship and satisfy the other requirements of the section. E Delete June as co-inventor by filing an Application Data Sheet in accordance with 37 C.F.R. 1.76, together with the required fee.

Delete June as co-inventor by filing an Application Data Sheet in accordance with 37 C.F.R. 1.76, together with the required fee. - think: june is just like a tech, or nay employee at a company. not actually an inventor

Which of the following statements relating to design patents does not accord with proper USPTO practice and procedure? A Both design and utility patents may be obtained on an article if the invention resides both in its utility and ornamental appearance. B The design for an article consists of the visual characteristics embodied in or applied to an article. C Design patent applications are included in the Patent Cooperation Treaty (PCT), and the procedures followed for PCT international applications are to be followed for design patent applications. D A claim directed to a computer-generated icon shown on a computer screen complies with the "article of manufacture" requirement of 35 USC 171. E A claimed design may encompass multiple articles or multiple parts within an article.

Design patent applications are included in the Patent Cooperation Treaty (PCT), and the procedures followed for PCT international applications are to be followed for design patent applications. - there is no PCT for designs - there is an actual international design patent application

A registered practitioner files an amendment to the client's claim which inserts language into the claim. The primary examiner improperly rejects the claim under 35 USC 112(a) description requirement. The examiner's rejection states that the amendment inserted new matter which does not have descriptive support in the original specification. The examiner correctly points out that there is no literal support for the amendatory claim language in the original specification, but erroneously concludes that it constitutes new matter. Assume that there is support for the amendment in the original disclosure. In accordance with the patent laws, rules and procedures as related in the MPEP, a proper reply would include which of the following argument(s) to show the examiner is in error? A The original specification would enable one of ordinary skill in the art to practice the invention as now claimed. B Literal support for new claim language is not required. C The original specification reasonably conveys to one of ordinary skill in the art that the inventor had the claimed invention in his/her possession as of the filing date of the application. D The new claim language is described in a related application filed by the inventor that is now a U.S. patent. E (B) and (C).

E (B) and (C).

our client John filed a provisional application about six months ago on an improved Bunsen burner. Now today your client tells you that he has an improvement to the invention that he wants to protect. Which of the following would be in accordance with current PTO rules and procedures? A File a continuation application under Rule 1.53(b) of the provisional and add the second improvement by preliminary amendment filed with the continuation. B File a second provisional directed to the second improvement. C File an application under Rule 1.53(d) with a specification including both the original and the second improvement and making the required reference to the provisional. D File a non-provisional application directed to both the original and the second improvement and making the required reference to the provisional. E (B) and (D)

E (B) and (D)

In 1995 Patent Agent filed a U.S. patent application containing five claims (Application 1). All five claims are fully supported under 35 USC 112 by the disclosure of Application 1. In 2000, Patent Agent filed a U.S. patent application (Application 2) that was a continuation-in-part of Application 1. Application 2 adds new subject matter to the disclosure of Application 1, and ten additional claims. Of the fifteen claims in Application 2, claims 1-5 are exactly the same as Application 1, claims 6-10 are fully supported under 35 USC 112 by the disclosure of Application 1, and claims 11-15 are fully supported under 35 USC 112 only by the newly added subject matter of Application 2. The effective filing date for claims in Application 2 is: A 1-15 is 2000. B 1-15 is 1995. C 1-10 is 1995. D 11-15 is 2000. E (C) and (D).

E (C) and (D). If the application is a continuation-in-part of an earlier U.S. application, any claims in the new application not supported by the specification and claims of the parent application have an effective filing date equal to the filing date of the new application. Any claims which are fully supported under 35 U.S.C. § 112 by the earlier parent application have the effective filing date of that earlier parent application

nventor Jones files an application under 35 USC 111(a) on March 27, 2002. The application is a continuation of an international application, which was filed on December 1, 2000. The international application claims priority to a U.S. provisional application filed December 2, 1999. The international application designated the United States, and was published in English under PCT Article 21(2). All applications contained the exact same disclosure. In accordance with the patent laws, rules and procedures as related in the MPEP, what, if any, is the earliest prior art date under 35 USC 102(e) for the publication of the 35 U.S.C. 111(a) application under 35 USC 122(b)? A None, the publication has no prior art date under 35 U.S.C. 102(e) B 27-Mar-02 C 11-Dec-01 D1-Dec-00 E 2-Dec-99

E 2-Dec-99

Igor filed a design patent application in the USPTO on January 24, 2000, which issued as a design patent on January 23, 2001. Igor's design patent covered a design that became immediately popular, resulting in numerous inquiries for licenses from various manufacturers. Igor would like to financially exploit his patent by licensing for five years. However, Igor has decided to dedicate five years of his patent term to the public. Which of the following is in accord with the USPTO rules and the procedures set forth in the MPEP, while best allowing Igor to pursue his intentions? A Record in the USPTO an assignment of all right, title, and interest in the patent to the public, conditioned on the receipt by Igor of all royalties from licensing the patent after the first five years of the patent term. B File a disclaimer in the USPTO dedicating to the public the first five years of the patent term. C File a disclaimer in the USPTO dedicating to the public that portion of the term of the patent from January 24, 2015 to January 24, 2020. D File a disclaimer in the USPTO dedicating to the public half of all royalties received from licensing the patent for the terminal part of the term of the patent. E File a disclaimer in the USPTO dedicating to the public that portion of the term of the patent from January 24, 2010 to January 23, 2015.

E File a disclaimer in the USPTO dedicating to the public that portion of the term of the patent from January 24, 2010 to January 23, 2015. NOT C because design patents only last fifteen years

On January 2, 2016, a registered practitioner filed a patent application with the USPTO for inventor Bock. The application includes a specification and a single claim to the invention, which reads as follows: 1. A new string consisting only of material Z that has the ability to stretch to beyond its initial unstretched length. On June 2, 2016, the practitioner received an Office action from the primary examiner rejecting the claim. The claim is solely rejected under 35 USC 102 in view of Patent A, which discloses a string consisting only of material Z. The Office action states, "Patent A discloses a string consisting only of material Z. Patent A does not expressly teach the stretchability property of the string. Nevertheless, the recited stretchability is inherent in the string of patent A. Accordingly, patent A anticipates the claimed string." Mr. Bock believes he is entitled to a patent to his new string and authorizes the practitioner to reply to the Office action by arguing that his string stretches to ten times its initial unstretched length, something that patent A does not teach. Since this is not expressly taught in Patent A, the practitioner argues, Patent A cannot anticipate the claimed string. In accordance with USPTO rules and procedures set forth in the MPEP, is the practitioner's reply persuasive as to error in the rejection? A Yes. B Yes, but the claim should now be rejected again, this time under 35 USC 103 as obvious over Patent A. C Yes, because the stretchability property is expressly taught by Patent A. D Yes, examiner nowhere addresses the claimed limitation of stretching the string beyond its initial unstretched length. E No.

E No. Something Which Is Old Does Not Become Patentable Upon The Discovery Of A New Property," "claiming of a new use, new function or unknown property which is inherently present in theprior art does not necessarily make the claim patentable.

Determine which of the following documents, if any, must also contain a separate verification statement in accordance with the patent laws, rules and procedures as related in the MPEP. A A request to correct inventorship in a pending application. B A petition to make an application special. C A claim for foreign priority. D A substitute specification. E None of the above.

E None of the above.

Which of the following results in abandonment of the invention? A An express abandonment of a pending application. B Failure to take appropriate action in a pending application. C Failure to pay the issue fee. D Passage of more than twelve months in the case of a provisional application. E None of the above.

E None of the above. **all options will abandon the APPLICATION, but not the INVENTION

Which of the following establishes a statutory bar under 35 USC 102 to patentability of Applicant's claimed invention? A To further develop the invention, Applicant's invention was tested and experimented with in the United States more than one year prior to applicant's effective U.S. filing date, but the invention at the time was not fit for its intended purpose and important modifications concerning the claimed features resulted from the experimentation. The first actual reduction to practice occurred after the effective U.S. filing date. B Applicant's invention was sold in a WTO member country outside the United States less than one year prior to applicant's effective U.S. filing date, and the sale was merely market testing of the invention to determine product acceptance. C Applicant's invention is rendered obvious by the combination of two U.S. patents, both of which were patented more than one year prior to applicant's effective filing date. D Applicant's invention was sold outside the United States in a non-WTO member country, less than one year prior to applicant's effective U.S. filing date, but the sale was merely an attempt at market penetration. E None of the above.

E None of the above. not A because it is permitted experimental testing

An application naming X and Y as joint inventors, filed on April 3, 2018, has a single pending claim, and does not claim the benefit of any earlier application. Which, if any, of the following items of prior art that have been relied on in various rejections of the claim may be overcome by a suitable affidavit under 37 CFR 1.130 in accordance with the patent laws, rules and procedures as related in the MPEP? A A U.S. patent to G that issued on March 27, 2017, has an effective U.S. filing date of January 4, 2015, and does not claim the "same patentable invention" (as defined in 37 CFR 1.601(n)) as the rejected claim. B A U.S. patent to P that issued on June 5, 2018, has an effective U.S. filing date of February 1, 2016, and includes a claim that is identical to the rejected claim. C A journal article to H published on December 10, 2017, and characterized in the application being examined as "describ[ing] the prior art." D A foreign patent issued to X and Y on November 7, 2016, which claims the same subject matter as the rejected claim and is based on an application filed on January 3, 2015. E None of the above.

E None of the above. why is it not C? because you cannot file an affadvit to swear behind something you've already admitted to be prior art

While traveling through Germany (a WTO member country) in December 2009, Thomas (a Canadian citizen) conceived of binoculars for use in bird watching. The binoculars included a pattern recognition device that recognized birds and would display pertinent information on a display. Upon Thomas' return to Canada (a NAFTA country) in January 2010, he enlisted his brothers Joseph and Roland to help him market the product under the tradename "Birdoculars." On February 1, 2010, without Thomas' knowledge or permission, Joseph anonymously published a promotional article written by Thomas and fully disclosing how the Birdoculars were made and used. The promotional article was published in the Saskatoon Times, a regional Canadian magazine that is also widely distributed in the United States. Thomas first reduced the Birdoculars to practice on March 17, 2010 in Canada. A United States patent application properly naming Thomas as the sole inventor was filed September 17, 2010. That application has now been rejected as being anticipated by the Saskatoon Times article. Which of the following statements is most correct? A Thomas can rely on his activities in Canada in establishing a date of invention prior to publication of the Saskatoon Times article. B In a priority contest against another inventor, Thomas can rely on his activities in Canada in establishing a date of invention. C In a priority contest against another inventor, Thomas can rely on his activities in Germany in establishing a date of invention. D Statements (A) and (B) are correct, but statement (C) is incorrect. E Statements (A), (B), and (C) are each correct.

E Statements (A), (B), and (C) are each correct. Thomas may rely on activities in both Germany (a WTO member country) and Canada (a NAFTA country) in establishing a date of invention prior to publication of the Saskatoon Times article or in establishing priority

In accordance with the MPEP and USPTO rules and procedure, correspondence transmitted to the USPTO by facsimile is not permitted in certain situations. Which of the following facsimile transmissions to the USPTO will be accorded a date of receipt by the USPTO? A Facsimile transmission of a request for reexamination under 37 CFR 1.510 or 1.913. B Facsimile transmission of drawings submitted under 37 CFR 1.81, 1.83 through 1.85, 1.152, 1.165, 1.174, or 1.437. C Facsimile transmission of a response to a Notice of Incomplete Nonprovisional Application for the purpose of obtaining an application filing date. D Facsimile transmission of a correspondence to be filed in a patent application subject to a secrecy order under 37 CFR 5.1 through 5.5 and directly related to the secrecy order content of the application. E Facsimile transmission of a continued prosecution application under 37 CFR 1.53(d) and an authorization to charge the basic filing fee to a deposit account.

Facsimile transmission of a continued prosecution application under 37 CFR 1.53(d) and an authorization to charge the basic filing fee to a deposit account. note: question is asking under which situation CAN you fax. You CAN fax CPA applications..you CANNOT fax any of the other options

Prior to filing a patent application for a client, a registered practitioner determined that the client was entitled to claim small entity status under 37 CFR 1.27. The practitioner filed a patent application for the client on November 1, 2016 together with a claim for small entity status under 37 CFR 1.27. On December 2, 2016, a Notice to File Missing Parts was mailed setting a two month period for reply and requiring the basic filing fee and the surcharge under 37 CFR 1.16(e). The practitioner timely submitted the small entity fees for the basic filing fee and the surcharge as required in the Notice. Shortly thereafter, the practitioner discovered that on October 31, 2016, the day before the application was filed, the client, without advising the practitioner, had assigned all rights in the invention that is the subject of the application to an entity that would not qualify for small entity status under 37 CFR 1.27. In accordance with the USPTO rules and the procedures set forth in the MPEP, which of the following actions would be the best action for the practitioner to take? A File a continuing application under 37 CFR 1.53(b) with the large entity filing fee and then file a letter of express abandonment under 37 CFR 1.138 in the original application after the continuing application has been accorded a filing date. B Promptly file a notification of loss of small entity status under 37 CFR 1.27(g) and, thereafter, pay large entity fees whenever any subsequent fees are required. C Wait until a Notice of Allowance is received and then timely submit the large entity issue fee along with a notification of loss of small entity status under 37 CFR 1.27(g). D File a paper under 37 CFR 1.28(c) requesting that the good faith error in claiming small entity status be excused and complying with the separate submission and itemization requirements of 37 CFR 1.28(c) and including payment of the deficiency owed. E Pay the difference between the large entity filing fee and small entity filing fee and the difference between the large entity surcharge and small entity surcharge within two months from the mail date of the Notice to File Missing Parts.

File a paper under 37 CFR 1.28(c) requesting that the good faith error in claiming small entity status be excused and complying with the separate submission and itemization requirements of 37 CFR 1.28(c) and including payment of the deficiency owed. Correcting Errors In Small Entity Status," states "37 CFR 1.28(c) provides that if small entity status is established in good faith and the small entity fees are paid in good faith, and it is later discovered that such status as a small entity was established in error or through error the Office was not notified of a change of status, the error will be excused upon compliance with the separate submission and itemization requirements

Where subject matter for which there is an enabling disclosure, but is not shown in the drawing or described in the detailed description preceding the claim(s), which of the following is not in accordance with the provisions of the MPEP? A In establishing a disclosure, applicant may rely not only on the description and drawing as filed but also on the original claims to show compliance with 35 USC 112(a). B Where subject matter not shown in the drawing or described in the description is claimed in the application as filed, and such original claim itself constitutes a clear disclosure of this subject matter, then the claim should be treated on its merits, and the applicant should be required to amend the drawing and description to show this subject matter. C If subject matter appearing in the original claim is not found in the drawing or detailed description, the claim should be rejected for noncompliance with 35 USC 112(a). D If the subject matter found in the claim is lacking in the drawing or detailed description, it is the drawing and description that are defective, not the claim. E The subject matter found in the original claims, but lacking in the drawing or detailed description, must be sufficiently specific and detailed to support an amendment of the drawing and detailed description.

If subject matter appearing in the original claim is not found in the drawing or detailed description, the claim should be rejected for noncompliance with 35 USC 112(a). It is not the claim that is defective, the description and drawing are, therefore the claim should not be objected nor rejected, the drawing/description should

Xavier files a complete first reply exactly 2 months after the mailing date of a final Office action which sets a 3 month shortened statutory period for reply. An Examiner's Amendment is necessary for the purpose of placing the application in condition for allowance. Which of the following statements is true? A If the Examiner's Amendment is mailed exactly 5 months after Xavier's reply, the application will be allowed. B The Examiner's Amendment must be made within the 3 month shortened statutory period of the final Office action to avoid abandonment of the application. C If the Examiner's Amendment is made exactly 4 months after Xavier's reply, the application will be allowed. D The Examiner's Amendment may be made at any time within 6 months of Xavier's reply to avoid abandonment. E Abandonment of the application will be avoided if Xavier accompanies his reply with a request for extension of time accompanied by the proper fee and the Examiner's Amendment is made within 6 months of Xavier's reply.

If the Examiner's Amendment is made exactly 4 months after Xavier's reply, the application will be allowed. Where a complete first reply to a final Office action has been filed within 2 months of the final Office action, an examiner's amendment to place the application in condition for allowance may be made without the payment of extension fees even if the examiner's amendment is made more than 3 months from the date of the final Office action. Not D, because this would cause abandonment

On January 3, 2016, a registered practitioner filed a continuation application that includes a benefit claim to a prior-filed application. The practitioner simultaneously filed in the prior-filed application an express abandonment in favor of a continuing application. The prior application contained five drawing figures described in the specification. However, the continuation application contains only four of the five drawing figures. The specification of the continuation application did not include a complete description of the missing drawing figure. A postcard from the USPTO, listing the contents of the continuation application, contains a note that only four drawing figures were received. The practitioner inadvertently omitted one of the drawing figures mentioned in the specification when he filed the continuation application. The missing drawing figure shows a claimed feature of the invention. On February 10, 2016, the practitioner received a Notice of Omitted Item(s) properly according a filing date of January 3, 2016 for the continuation application without the missing drawing figure and notifying the applicant that the drawing is missing. Which of the following procedures for filing the missing drawing would comply with the patent laws, rules and procedures as related in the MPEP for according the continuation application a January 3, 2016 filing date with the five drawing figures that were present in the application? A The practitioner files the missing drawing figure in response to the Notice of Omitted Item(s) within the time period set forth in the notice with no change in filing date. B The practitioner files the missing drawing figure and an amendment to the specification to add a complete description of the missing drawing figure in response to the Notice of Omitted Item(s) within the time period set forth in the notice. C The practitioner files an amendment to cancel the description of the missing drawing figure from the specification of the continuation application. D If the continuation application as originally filed includes an incorporation by reference of the prior-filed application to which the benefit is claimed, the practitioner can file the missing drawing figure any time prior to the first Office action. E (A) and (D)

If the continuation application as originally filed includes an incorporation by reference of the prior-filed application to which the benefit is claimed, the practitioner can file the missing drawing figure any time prior to the first Office action. *NOT A because if you do A your filing date we be the date you filed the missing figure. Remember, we are trying to find an option that retains our original filing date.

Assume that conception of applicant's complex pre-AIA invention occurred prior to the date of the reference, but reduction to practice occurred after the date of the reference. Which of the following is sufficient to overcome the reference in accordance with proper USPTO practice and procedure? A In a 37 CFR 1.131 affidavit or declaration, it is sufficient to allege that applicant or patent owner has been diligent. B In a 37 CFR 1.131 affidavit or declaration, it is sufficient to clearly establish conception of the invention prior to the effective date of the reference, and diligence from just prior to the effective date of the reference to actual reduction to practice of an invention and the filing of an application thereon is not relevant. C In a 37 CFR 1.131 affidavit or declaration, it is sufficient to clearly establish conception of the invention prior to the effective date of the reference. Diligence need not be considered. D In a 37 CFR 1.131 affidavit or declaration, it is sufficient to show conception and reduction to practice in any country. E In a 37 CFR 1.131 affidavit or declaration, it is always sufficient to prove actual reduction to practice for all mechanical inventions by showing plans for the construction of the claimed apparatus.

In a 37 CFR 1.131 affidavit or declaration, it is sufficient to clearly establish conception of the invention prior to the effective date of the reference, and diligence from just prior to the effective date of the reference to actual reduction to practice of an invention and the filing of an application thereon is not relevant.

In accordance with the MPEP and USPTO rules and procedure, an application for patent may be made on behalf of a joint inventor in certain situations. Who, by petition, may make application on behalf of a joint inventor who has refused to sign the application ("nonsigning inventor"), if the other joint inventor ("signing inventor") executes the application? (A) A person other than the signing inventor, to whom the nonsigning inventor has assigned the invention. (B) A person other than the signing inventor, with whom the nonsigning inventor has agreed in writing to assign the invention. (C) The signing inventor. D) A person other than the signing inventor, who shows a strong proprietary interest in the invention. (E) All of the above.

NOTE: in 2002 the answer on USPTO website was (C) The signing inventor, but now PLI says post-AIA it is (E) All of the above.

The Office mailed an Office action containing a proper final rejection dated July 8, 2016. The Office action did not set a period for reply. On January 7, 2017, in reply to the final rejection, a registered practitioner filed a request for continued examination under 37 CFR 1.114, a request for a suspension of action under 37 CFR 1.103(c) to suspend action for three months, and proper payment all required fees. No submission in reply to the outstanding Office action accompanied the request for continued examination. No other paper was submitted and no communication with the Office was held until after Midnight, January 8, 2017. Which of the following statements accords with the USPTO rules and the procedures set forth in the MPEP? A If an appropriate reply is submitted within the three month period of suspension permitted under 37 CFR 1.103(c), the application will not be held abandoned. B The application will not be held abandoned if an appropriate reply is submitted within the three month period of suspension and it is accompanied by a showing that the reply could not have been submitted within the period set in the final rejection. For example, the reply includes a showing based on an experiment that required 8 months to conduct. C No reply will prevent the application from being held abandoned. D If, on January 10, 2017, the primary examiner and applicant agree to an examiner's amendment that places the application in condition for allowance and a notice of allowance is mailed within the three month period of suspension, application X will not be held abandoned. E No other submission by applicant is necessary because application X is still pending. The examiner is required to act on the request for continued examination after expiration of the three month period of suspension.

No reply will prevent the application from being held abandoned. This is because the RCE filing was improper (not accompanied with anything), so it will not be recognized by the office and will be abandoned. Also note that if the office does not set a reply period for final rejection, you have a maximum of 6 months to respond still

A registered practitioner filed a first patent application wherein claims 1-10 claims are directed to a widget and claims 11-20 are directed to a method of making a widget. Following a proper restriction requirement, claims 1-10 were elected for prosecution. The primary examiner rejected claims 1-10. The practitioner filed a reply that only consisted of argument. The examiner was unpersuaded by the argument, and entered a final rejection of claims 1-10. In reply, the practitioner filed a continuing application containing claims 1-10 directed to a widget, and claims 11-20 directed to a method of using a widget. In the continuing application, the examiner enters a new written restriction requirement requiring a provisional election between claims 1-10 and claims 11-20. The practitioner believes the new restriction requirement is improper and would like the rejection in the parent application reviewed as well. The new restriction requirement has not been made final. Which of the following best describes whether and why, in accordance with the patent laws, rules, and procedures as related by the MPEP, the reply to the restriction requirement may be by appeal to the PTAB? A Yes. An immediate appeal to the Board can be filed to review the restriction requirement if any claims have been twice rejected. B No. An immediate appeal cannot be filed to the Board because the new claims directed to a method of using a widget have not been twice rejected. C Yes. An immediate appeal can be filed for any claims that have been twice rejected because the Board can also review any second restriction requirement made against the same claims. D No. An immediate appeal to the Board cannot be lodged because a provisional election has not been made of either the claims to a widget or claims to a method of use of the widget. E No. An immediate appeal cannot be taken because no claims are currently under rejection. Review of a final restriction requirement is only possible as a petitionable matter before a Technology Center Director. It is not an appealable matter to the Board.

No. An immediate appeal cannot be taken because no claims are currently under rejection. Review of a final restriction requirement is only possible as a petitionable matter before a Technology Center Director. It is not an appealable matter to the Board. NOTE: this question is asking you to respond to the restriction requirement, NOT the rejected claims. you would appeal the claim rejecitons, but petition the restriction requirement

n accordance with the USPTO rules and the procedures set forth in the MPEP, in which of the following cases is the date of actual receipt by the USPTO not accorded as the application filing date? A Provisional application filed without claims. B Non-provisional application filed containing an error in inventorship. C Non-provisional application filed which fails to identify the inventor(s). D Non-provisional application filed "by reference" where a complete copy of the reference filing is not submitted within 4 months of filing, or sixteen months from the priority date. E Non-provisional application filed using a certificate of mailing in accordance with 37 CFR 1.8.

Non-provisional application filed "by reference" where a complete copy of the reference filing is not submitted within 4 months of filing, or sixteen months from the priority date. Failure to identify inventorship or errors in inventorship does not affect filing date

A patent application is filed disclosing and claiming a system for detecting expired parking meters. The specification fully supports the original, sole claim. The application discloses that the "electronics control unit" contains a comparator and an alarm. The application includes several drawings. One of the drawings shows a block diagram of the system, illustrating the electronics control unit as a box, labeled "electronics control unit." The sole claim of the application is as follows:The claim. A system for detecting expired parking meters, comprising: a timer mechanism; an infrared sensor for detecting the presence of a parked vehicle; and an electronics control unit, including a comparator and an alarm, coupled to the infrared sensor and the timer mechanism.A final Office action, dated February 3, 2016, indicates that the sole claim contains allowable subject matter, but includes an objection to the specification, on the grounds that the subject matter of the electronics control unit, though described in a sufficiently specific and detailed manner in the original specification, was required to be shown in the drawings under 37 CFR 1.83. The Office action did not set a period for reply. Determine which of the following actions, if any, comports with the patent laws, rules and procedures as related in the MPEP for overcoming the objection. A On April 1, 2016, a Notice of Appeal is filed together with appropriate fees, and a brief pointing out that a patent should issue since the subject matter of the electronics control unit was adequately described in the original specification B On April 1, 2016, a drawing is filed in the USPTO illustrating only the comparator and alarm of the electronics control unit that was described in the original specification. C On April 1, 2016, a Notice of Appeal of appeal is filed together with appropriate fees, and a brief pointing out that the addition of a drawing showing the electronics control unit would not constitute addition of new matter since the electronics control unit was adequately described in the original specification. D On September 1, 2016, a petition is filed urging that no further drawing should be required because the subject matter of the electronics control unit, for purposes of the application, was adequately disclosed in the block diagram drawing. E None of the above.

On April 1, 2016, a drawing is filed in the USPTO illustrating only the comparator and alarm of the electronics control unit that was described in the original specification. Why is it not D? (D) is incorrect because the petition, filed more than six months after the final rejection, is not timely.

Ben hires a registered practitioner to prosecute his patent application. The practitioner drafted an application having fifteen claims. Claim 1 is independent, and each of claims 2-15 are singularly dependent upon claim 1. A proper non-final Office action is mailed to the practitioner. Following consultation with Ben, the practitioner timely prepared, signed, and filed a reply to the Office action containing an amendment that does not add new matter, but does add claims 16-27. Each of claims 16-27 is directed to the same invention sought to be patented through claims 1-15. The dependency of each of claims 16-27 reads "any of claims 5-15." For purposes of fee calculation in accordance with the patent laws, rules and procedures as related in the MPEP, how many total claims are contained in the application after the amendment is entered? A One hundred thirty-six. B One hundred thirty-five. C Twenty-seven. D One hundred forty-seven. E Fifteen.

One hundred forty-seven.

In which of the following situations, considered independently of each other, is the original, new, or amended claim supported in the application as filed? A An amendment to the specification changing the definition of "holder" from "is a hook" to "is a hook, clasp, crimp, or tong" and no amendment is made of the claim, which uses the term "holder." The amendment is filed one month after the application was filed. There was no previous supporting disclosure in the specification of the holder being a clasp, crimp, or tong. B An amendment to the specification and claims changing the definition of "holder" from "is a hook" to "is a hook, clasp, crimp, or tong." The amendment is filed one month after the application was filed. There was no previous supporting disclosure in the specification of the holder being a clasp, crimp, or tong. C Original claim 1 in the application refers to "a holder," and original claim 2 depends from and refers to claim 1 stating, "said holder is a hook, clasp, crimp, or tong." There is no disclosure in the specification preceding the claims in the application as filed for the holder to be a clasp, crimp, or tong. D An amendment is filed presenting a claim to an electrical insulating device, copied from a patent for the purpose of provoking an interference. The claim refers to "nonconductive plastic holder." The application as filed contains a broad generic disclosure describing electrical insulating devices. The holder is described in the specification of the application as "conducting electricity." There is no disclosure in the specification of the holder being "nonconductive." E All of the above.

Original claim 1 in the application refers to "a holder," and original claim 2 depends from and refers to claim 1 stating, "said holder is a hook, clasp, crimp, or tong." There is no disclosure in the specification preceding the claims in the application as filed for the holder to be a clasp, crimp, or tong.

The USPTO notifies John, a registered patent agent who is representing applicant A, that after a reasonable search, the USPTO has been unable to locate applicant A's patent application. By which of the following procedures may John avoid abandonment of applicant A's application within the time period set by the USPTO? A Provide the USPTO with a copy of his record of all the correspondence between his office and the USPTO, assuming the existence of such record. B Provide the USPTO with a list of all the correspondence between his office and the USPTO, assuming the existence of such list, and a statement that the list is complete and accurate. C Provide the USPTO with a statement that he does not possess any record of the correspondence between his office and the USPTO because his files were destroyed. D Provide the USPTO with a record of all the correspondence between his office and the USPTO, and a statement that the papers produced are his complete record of all the correspondence between his office and the USPTO, assuming the existence of such record. E Provide the USPTO with a copy of his record of all the correspondence between his office and the USPTO, assuming the existence of such record, a list of all such correspondence, and a statement that he is not aware of any correspondence between his office and the USPTO that is not among his records.

Provide the USPTO with a statement that he does not possess any record of the correspondence between his office and the USPTO because his files were destroyed.

If a prima facie case of obviousness is properly established by a primary examiner, how can an applicant effectively rebut the rejection in accordance with proper USPTO practice and procedure? A Rebuttal may be by way of arguments of counsel used in place of factually supported objective evidence to rebut the prima facie case. B Rebuttal may be by way of an affidavit or declaration under 37 CFR 1.132 containing objective evidence arising out of a secondary consideration related to the claimed invention. C No substantive showing is required by applicant. The burden remains on the examiner to maintain a prima facie case. D Rebuttal evidence must be found elsewhere than in the specification. E Rebuttal may be by way of arguing that the prior art did not recognize latent properties.

Rebuttal may be by way of an affidavit or declaration under 37 CFR 1.132 containing objective evidence arising out of a secondary consideration related to the claimed invention. Affidavits or declarations containing objective evidence of criticality, unexpected results, commercial success, long-felt but unsolved needs, failure of others, skepticism of experts, is considered by an examiner *this is the only time secondary considerations are probably valid not E because finding that mere recognition of latent properties in the prior art does not render a known invention unobvious

Evidence that a claim may not comply with 35 USC 112(b) occurs in accordance with the USPTO rules and the procedure set forth in the MPEP where: A Remarks filed by applicant in a reply or brief regarding the scope of the invention differ and do not correspond in scope with the claim. B There is a lack of agreement between the language in the claims and the language set forth in the specification. C The scope of the claimed subject matter is narrowed during pendency of the application by deleting the originally much broader claims, and presenting claims to only the preferred embodiment within the originally much broader claims. D Claims in a continuation application are directed to originally disclosed subject matter (in the parent and continuation applications) which applicants did not regard as part of their invention when the parent application was filed. E All of the above.

Remarks filed by applicant in a reply or brief regarding the scope of the invention differ and do not correspond in scope with the claim.

A provisional U.S. patent application covering a unique chair was filed in the U.S. Patent Office on March 14, 2010. The application claims priority from a U.K. application filed April 30, 2009. The U.K. application was published on November 23, 2010. A Utility application claiming priority from the provisional application was filed in the U.S. Patent Office on September 25, 2010. An official action in the U.S. application was sent to the applicant on November 3, 2011 setting a shortened statutory period of three months to respond. No response was filed. A proper continuation of the utility application was filed on December 14, 2011, and issued as a U.S. patent on July 5, 2013. When will the patent expire, assuming all maintenance fees are timely paid?

Sept 25 2030

The claims in a patent application having been twice or finally rejected, the applicant files a timely Notice of Appeal on January 2, 2016. In accordance with USPTO rules and procedures set forth in the MPEP, which of the following situations should the USPTO not notify the applicant that the Appeal Brief is defective and allow him an opportunity to correct the deficiency? A The Appeal Brief is filed on July 10, 2016, without a request for extension of time under 37 CFR 1.136. B The Appeal Brief is submitted unsigned. C The Appeal Brief states that the claims do not stand or fall together, and presents argument as to why the claims are separately patentable, but the primary examiner does not agree with the applicant's argument. D The Appeal Brief does not state whether the claims stand or fall together, but presents arguments why the claims subject to the same rejection are separately patentable. E The Appeal Brief does not address any of the grounds of rejection stated by the primary examiner, has no argument section, and is non-compliant as to 37 CFR 41.37.

The Appeal Brief states that the claims do not stand or fall together, and presents argument as to why the claims are separately patentable, but the primary examiner does not agree with the applicant's argument. . If the examiner disagrees with the reasons given, the reason for disagreement should be addressed in the Examiner's Answer. If the examiner disagrees with appellant's statement in the brief that certain claims do not stand or fall together, the examiner explains in the examiner's answer why the claim grouping listed in the brief is not agreed with and why, if appropriate, e.g., the claims as listed by the appellant are not separately patentable.

Adams filed Application X on March 1, 2021. Beth filed application Y on May 1, 2021. Neither application has been published. Applications X and Y are copending and commonly assigned. Earlier filed application X claims the same invention as claimed in application Y using identical language. In accordance with the MPEP, which of the following actions should the examiner or assignee follow? A The claims to the same invention in application Y should be rejected under 35 USC 102(a)(1) as being anticipated by application X. B The claims to the same invention in application Y should be rejected under 35 USC 102(a)(2) as being anticipated by application X. C The claims to the same invention in application Y should be rejected under 35 USC 102(a)(2) as being provisionally anticipated by application X. D The common assignee should file a terminal disclaimer in application Y to avoid any question of double patenting. E The claims to the same invention in application Y should be rejected under 35 USC 101 as being anticipated by application X. Previous Next

The claims to the same invention in application Y should be rejected under 35 USC 102(a)(2) as being provisionally anticipated by application X. Why this and not D? remember, only nonstatutory double patenting can be overcome by filing a terminal disclaimer, where as statutory double patenting cannot be overcome this way (claims would have to be ammended). If applications are commonly assigned and the effective filing dates are different, then a provisional rejection of the later filed application should be made **provisional rejections applied to COPENDING applications having different effective filing dates wherein each application has a common assignee or a common inventor.

A primary examiner is examining a patent application. The application includes a specification and a single claim to the invention that reads as follows:1. A building material to be used as an alternative to brick in the construction of a house, said building material comprising compressed refuse, the majority of which is wood.In the specification, the inventor explains that the wood to be used in the inventive building material should be balsa wood. According to the specification, balsa-containing building material has the advantage of being lighter than brick. In a first Office action mailed to the registered practitioner representing the inventor the single claim was rejected as anticipated under 35 U.S.C. 102 over Patent A. Patent A issued more than one year before the effective filing date of the application, and teaches a building material to be used as an alternative to brick in the construction of a house comprising compressed refuse, the majority of which is pine. The practitioner replies to the first Office action by arguing that the invention is different from that of Patent A. According to the practitioner, the inventor uses balsa wood, not pine. The claim has not been amended. Which of the following describes how the examiner should proceed in accordance with the patent laws, rules and procedures as related in the MPEP? A The examiner should allow the claim. B The examiner should allow the claim only after including a Reasons for Allowance pointing out that the inventor argues that her invention is directed to using balsa wood, not pine. C The examiner should issue a Final Rejection again rejecting the claim as anticipated under 35 USC102 over Patent A. D The examiner should reopen prosecution and begin anew, this time searching for a reference that shows a building material containing balsa wood. E The examiner should withdraw the rejection but issue a new Office action this time rejecting the claim under 35 USC 112, second paragraph, because the claim is broad enough to encompass using pine

The examiner should issue a Final Rejection again rejecting the claim as anticipated under 35 USC102 over Patent A. - for a refere3nce to anticipate a claim, it must teach every element of the claim! the inventor never ammended the claim - and the claim is given it's broadest possible interpretation

Co-inventors Smith and Jones filed an application for a patent on a cell phone, on May 15, 2016. They received a first Office action from a primary examiner rejecting the claims under 35 USC 102(a)(1) over a publication by Bell and Watson, published on April 5, 2016, describing a cell phone having all the same features as is claimed in the patent application. In reply, the co-inventors each submitted a declaration under 37 CFR 1.131 stating that they had actually reduced the invention to practice no later than March 13, 2016. However, the declarations failed to include two claimed features. Neither the particular antenna needed to enable the cell phone could receive transmissions from the local cellular transmitting tower, nor a detachable carrying strap was included in the declarations. As evidence of their prior reduction to practice, Smith and Jones submitted their co-authored journal article. The journal article contained a figure of the cell phone as described in the declarations. That is, the cell phone shown in the figure of the article lacked an antenna and a detachable strap. The article was received by the journal on March 13, 2016, and was published on April 30, 2016. The cell phones shown in the figure in the Bell and Watson publication, and in the Smith and Jones patent application have the particular antenna and a detachable strap. Which of the following actions, if taken by the examiner, would be the most proper in accordance with the patent laws, rules and the procedures as related in the MPEP? A The examiner should maintain the rejection of the claims under 35 USC 102(a) and make the rejection final inasmuch as a 1.131 affidavit has no application in an AIA case. B The examiner should withdraw the rejection and look for references which have a publication date prior to May 15, 2015. C The examiner should withdraw the rejection and notify Smith and Jones that their application is in condition for allowance. D The examiner should maintain the rejection, but indicate that the claims would be allowable if Smith and Jones provided an original copy of the figure published in their journal article as factual support for their declarations. E The examiner should maintain the rejection and inform Smith and Jones that the declarations are insufficient because they cannot "swear behind" a reference which is a statutory bar. Previous Next

The examiner should maintain the rejection of the claims under 35 USC 102(a) and make the rejection final inasmuch as a 1.131 affidavit has no application in an AIA case.

Lars, an inventor living in Iceland, conceives of a new method of growing lichen and discloses the idea to his rich neighbor Eric. Eric tries the idea and, after it works, agrees with Lars to give Lars all the fish he can eat for the rest of his life in return for a one-half, undivided interest in the patent. Lars also agrees that Eric will pay for and handle filing applications in the U.S. and Norway. Eric then retains the only Icelandic patent attorney, Leif, who files an application in Iceland and who contacts you to file an application in the United States. Which of the following is true? A The inventor is Lars; Lars, Eric, and Leif each have a duty to disclose information to the Patent Office material to the examiner's decision to issue a patent. B The inventors are Lars and Eric; Lars, Eric, Leif and you each have a duty to disclose information to the Patent Office material to the examiner's decision to issue a patent. C The inventor is Lars; Eric, Lars, Leif and you each have a duty to disclose information to the Patent Office material to the examiner's decision to issue a patent. D The inventor is Lars; Lars and you each have a duty to disclose information to the Patent Office material to the examiner's decision to issue a patent. E The inventor is Lars; Lars has a duty to disclose information to the Patent Office material to the examiner's decision to issue a patent.

The inventor is Lars; Eric, Lars, Leif and you each have a duty to disclose information to the Patent Office material to the examiner's decision to issue a patent.

Shawn Slippery was granted a design patent directed to a troll design on May 23, 2005. After many unsuccessful attempts, he managed to convince a young woman he met in a bar that his patent was valuable and she agreed to buy the patent for $20,000 in cash, which she had secreted on her person. She gave Shawn the money, and Shawn wrote the following on a napkin in the bar, which Mary stuffed in her purse."I, Shawn Slippery hereby assign all rights in my troll design patent D 234,781 to Mary Wistful.Shawn Slippery June 3, 2005"Mary came to your office today and told you she was jogging yesterday with a friend who told her that Shawn had been trying to sell the same patent to others. While she has yet to use the patent in any way, she believes that someday it may be valuable. You should advise her that: A Every patent that successfully survives reexamination is strengthened by the experience and a reexamination should be filed. B The napkin cannot be recorded because it has to be on A4 paper and must be signed by Mary as well. C The napkin should be copied onto regular paper and that paper recorded. D She should keep her ears open and advise you immediately if Shawn sells the invention again. E She should immediately execute an assignment to you which you will prepare and which you will then record.

The napkin should be copied onto regular paper and that paper recorded. -Assignment is already executed

Which of the following statement(s) is in accordance with patent laws, rules and procedures as related in the MPEP regarding filing of an Application Data Sheet (ADS) in the USPTO? A All non-provisional applications must include an ADS when the application is originally filed. B If an ADS is filed at the same time as an oath or declaration under 37 CFR 1.63 or 1.67 and the information supplied in the two documents is inconsistent, the information provided in the ADS will always govern. C If an ADS is filed at the same time as an oath or declaration under 37 CFR 1.63 or 1.67 and the information supplied in the two documents is inconsistent, the oath or declaration will govern any inconsistency related the claiming of benefit under 35 USC 119(e), 120, 121 or 365(c). D If an ADS is filed after an oath or declaration under 37 CFR 1.63 or 1.67 is filed, and the information supplied in the two documents is inconsistent, the information provided in the ADS will always govern. E The oath or declaration under 37 CFR 1.63 or 1.67 governs inconsistencies with the ADS when the inconsistency concerns setting forth the name(s) of the inventor(s) under 35 USC 115.

The oath or declaration under 37 CFR 1.63 or 1.67 governs inconsistencies with the ADS when the inconsistency concerns setting forth the name(s) of the inventor(s) under 35 USC 115. because an ADS is a voluntary submission in either a provisional application or a nonprovisional application.

John, unaware of the existence of Jane's U.S. patent, which issued on Tuesday, July 11, 2012, files a patent application on Friday, January 11, 2013. John's application and Jane's patent are not commonly owned. On Thursday, July 11, 2013, in reply to an Office action rejecting all of his claims, John files an amendment canceling all of his claims and adding claims setting forth, for the first time, "substantially the same subject matter" as is claimed in Jane's patent. The examiner rejects John's claims on the basis of 35 USC 135(b). Which of the following statements accords with the USPTO rules and the procedures set forth in the MPEP? A The rejection is improper because 35 USC 135(b) relates to interferences. B The rejection is proper because 35 USC 135(b) is not limited to inter partes proceedings, but may be used as a basis for ex parte rejections. C Since John's claims would interfere with Jane's unexpired patent, the proper procedure is for the examiner to declare an interference rather than to reject John's claims. D The rejection is proper merely by reason of the fact that John's claims are broad enough to cover the patent claims. E The rejection is improper inasmuch as John is claiming "substantially the same subject matter" as is claimed in the patent.

The rejection is proper because 35 USC 135(b) is not limited to inter partes proceedings, but may be used as a basis for ex parte rejections.

A product-by-process claim is properly rejected over a reference under 35 USC 102. Which of the following statements is incorrect? A There is no anticipation unless each of the process steps recited in the claim is disclosed or inherent in the applied reference. B If the applied reference reasonably indicates that a product disclosed therein is the same or substantially the same as the claimed product, the burden shifts to the applicant to provide evidence to the contrary. C The rejection cannot be overcome by evidence of unexpected results. D The rejection can be overcome by evidence that the product in the reference does not necessarily or inherently possess a characteristic of the applicant's claimed product. E An affidavit or declaration under 37 CFR 1.131 cannot overcome a proper rejection under pre-AIA 35 USC 102(b) over a reference.

There is no anticipation unless each of the process steps recited in the claim is disclosed or inherent in the applied reference. **The patentability of a product-by-process claim is determined based on the product itself, not on the process of making it.

Evidence of unexpected results is relied upon to overcome a prima facie case of obviousness. Which of the following is incorrect? A The evidence must compare the claimed invention to the closest prior art. B The evidence must be commensurate in scope with the claims. C Data relied upon to show unexpected results need not cover the full range of the claims if one of ordinary skill in the art could ascertain a trend in the data that would allow that person to reasonably extend the probative value of the data to the full scope of the claims. D Unexpected results can be shown by factual evidence or, if no factual evidence is available to the applicant, by sound argument by the applicant's agent or attorney. E The evidence need not be in an affidavit or declaration under 37 CFR 1.132 if the evidence is presented in the specification of an application to which the applicant has attested.

Unexpected results can be shown by factual evidence or, if no factual evidence is available to the applicant, by sound argument by the applicant's agent or attorney. (D) is the most correct answer because mere attorney argument, unsupported by factual evidence, is insufficient to establish unexpected results.

A registered practitioner properly recorded an assignment document for application A identifying XYZ Company as the assignee. The document assigns to XYZ Company the "subject matter claimed in Application A." A proper restriction requirement was made by a primary examiner in application A between two distinct inventions, and the practitioner elected to prosecute one of the inventions. Application A was prosecuted, and later became abandoned. Before the abandonment date of application A, the practitioner filed a complete application B as a proper divisional application of application A. Application B claimed the nonelected invention of Application A, and was published as a U.S. application publication. XYZ Company remains the assignee of application A. What must the practitioner do in accordance with the patent laws, rules and procedures as related in the MPEP to ensure that XYZ Company is listed as the assignee on the face of any patent issuing from application B? A File a proper assignment document in application B identifying XYZ Company as the assignee. B File a proper assignment document in application B identifying XYZ Company as the assignee, and confirm that USPTO's bibliographic data for application B identifies XYZ Company as the assignee by checking the filing receipt for application B, the U.S. application publication of application B, or the USPTO's Patent Application Information Retrieval (PAIR) system data for application B, depending on when the practitioner filed the assignment document in application B. C Confirm that XYZ Company is identified as the assignee on the U.S. application publication of application B. D File a proper assignment document in application B identifying XYZ Company as the assignee, and confirm that XYZ Company is identified as the assignee on the U.S. application publication of application B. E Upon allowance of application B, the practitioner must identify XYZ Company as the assignee in the appropriate space on the Issue Fee Transmittal form for specifying the assignee for application B.

Upon allowance of application B, the practitioner must identify XYZ Company as the assignee in the appropriate space on the Issue Fee Transmittal form for specifying the assignee for application B. In the case of a division or continuation application, a prior assignment recorded against the original application is applied to the division or continuation application because the assignment recorded against the original application gives the assignee rights to the subject matter common to both applications. the patent issues to the assignee if so indicated on the IssueFee Transmittal form

Applicant received a Final Rejection with a mail date of Tuesday, February 29, 2016. The Final Rejection set a 3-month shortened statutory period for reply. Applicant files an Amendment and a Notice of Appeal on Monday, March 27, 2016. The examiner indicates in an Advisory Action that the Amendment will be entered for appeal purposes, and how the individual rejection(s) set forth in the final Office action will be used to reject any added or amended claim(s). The mail date of the examiner's Advisory Action is Wednesday, May 31, 2016. In accordance with the USPTO rules and the procedures set forth in the MPEP, which of the following dates is the last date for filing a Brief on Appeal without an extension of time? A Saturday, May 27, 2016. B Monday, May 29, 2016 (a Federal holiday, Memorial Day). C Tuesday, May 30, 2016. D Wednesday, May 31, 2016. E Tuesday, August 29, 2016.

Wednesday, May 31, 2016. - normally, the appeal brief is due 2 months from the notice of appeal. However, there is the "2 month rule" in play here, If an applicant replies within 2 months from the date of a final rejection which had set a 3 month statutory period for reply AND the office did not mail an advisory action until the end of the 3 month statutory period, the period for reply is 2 months from the advisory action

n accordance with the USPTO rules and the procedures set forth in the MPEP, impermissible recapture in an application exists ________________________. A if the limitation now being added in the present reissue was originally presented/argued/stated in the original application to make the claims allowable over a rejection or objection made in the original application. B if the limitation now being omitted or broadened in the present continuation was originally presented/argued/stated in a parent application to make the claims allowable over a rejection or objection made in the parent application. C if the limitation now being omitted or broadened in the present reissue was originally presented/argued/stated in the original application to make the claims allowable over a rejection or objection made in the original application. D if the limitation now being omitted or broadened in the present reissue was being broadened for the first time more than two years after the issuance of the original patent. E None of the above.

if the limitation now being omitted or broadened in the present reissue was originally presented/argued/stated in the original application to make the claims allowable over a rejection or objection made in the original application. **(look for - omitted/broadened/reissue and presented/argued/stated)

In accordance with patent laws, rules and procedures as related in the MPEP, an abandoned U.S. patent application: A is never available as evidence of prior art. B may become prior art only when it is properly incorporated by reference in the disclosure of a U.S. patent. C may become prior art as of its filing date, but only if it is properly incorporated by reference in the disclosure of a U.S. patent. D may become evidence of prior art as of its filing date, but only if it is properly incorporated by reference in the disclosure of a U.S. patent or U.S. application publication. E may become prior art when it is properly incorporated by reference in the disclosure of a U.S. application publication.

may become prior art when it is properly incorporated by reference in the disclosure of a U.S. application publication. ANSWER IS E: watch out for "only" in the other wording

Which of the following is the best way to recite a claim limitation so that it will be interpreted by the examiner in accordance with 35 USC 112(f)? A dot matrix printer for printing indicia on a first surface of a label. B dot matrix printer means coupled to a computer. C means for printing indicia on a first surface of a label. D printer station for printing indicia on a first surface of a label. E All of the above.

means for printing indicia on a first surface of a label. Note - we are talking about claim limitations..limitation must use the phrase "means for",

In accordance with MPEP § 1500, relating to design patent applications: A the invention may be properly represented in a single application by both an ink drawing and a black and white photograph. B the invention may be properly represented in a single application by a black and white photograph disclosing environmental structure by broken lines, in lieu of an ink drawing if the invention is shown more clearly in the photograph. C the invention may be properly represented in a single application by both an ink drawing and a color photograph, and the application should be accompanied by the required petition, fee, three sets of color photographs, and an amendment to the specification. D the invention may be properly represented by a color photograph disclosing environmental structure by broken lines, in lieu of an ink drawing if the invention is not capable of being illustrated in an ink drawing. E the invention may be properly represented by a color photograph if the invention is not capable of being illustrated in an ink drawing, and if the application is accompanied by an amendment to the specification is presented to insert required language regarding the color photographs, and three sets of color photographs.

the invention may be properly represented by a color photograph if the invention is not capable of being illustrated in an ink drawing, and if the application is accompanied by an amendment to the specification is presented to insert required language regarding the color photographs, and three sets of color photographs.


संबंधित स्टडी सेट्स

entrepreneurial finance Exam 1 Chapters 1-5

View Set

FOXYLEARNING- Intro to Verbal Behavior

View Set

World History - Unit 4: assignment questions quiz 2

View Set

Parts of the Brain & what they control

View Set

Exam 2 EAQs: Ch. 36 - Valvular Heart Disease and Cardiomyopathy

View Set

#2_Fall Protection_Video Questions_OSHA 30-HR

View Set

AP Psychology: Schools of Psychology

View Set

Chapter 8: Dissociative Disorders and Somatoform Disorders

View Set

King Menes of Egypt (The 1st Pharaoh)

View Set